學測數學 | 113學測數學A詳解 | 113學年度學科能力測驗數學A解析 - 傑森的數學網站

學測數學 | 113學測數學A詳解 | 113學年度學科能力測驗數學A解析

113學測數學詳解


第壹部分:選擇題(占 6 5 分)

一、單選題(占 3 0 分)

1.

研究顯示:服用某藥物後,在使用者體內的藥物殘留量隨時間呈指數型衰退。已知在服用某藥物 2 小時後,體內仍殘留有該藥物的一半劑量,試問下列哪一選項正確?

(1) 服用 3 小時後,體內仍殘留有該藥物的 $\dfrac{1}{3}$ 劑量
(2) 服用 4 小時後,體內仍殘留有該藥物的 $\dfrac{1}{4}$ 劑量
(3) 服用 6 小時後,體內仍殘留有該藥物的 $\dfrac{1}{6}$ 劑量
(4) 服用 8 小時後,體內仍殘留有該藥物的 $\dfrac{1}{8}$ 劑量
(5) 服用 10 小時後,體內仍殘留有該藥物的 $\dfrac{1}{10}$ 劑量

(2)

因為藥物殘留量隨時間呈指數型衰退,而且兩小時後的殘留量變為 $\dfrac{1}{2}$ ,故經過 $h$ 小時後,藥物殘留量為原本的 $\left(\dfrac{1}{2}\right)^{(\frac{h}{2})}$ 。
(1) $\left(\dfrac{1}{2}\right)^{(\frac{3}{2})}\neq \dfrac{1}{3}$ ❌
(2) $\left(\dfrac{1}{2}\right)^{(\frac{4}{2})}= \dfrac{1}{4}$ ✔
(3) $\left(\dfrac{1}{2}\right)^{(\frac{6}{2})}=\dfrac{1}{8}\neq \dfrac{1}{6}$ ❌
(4) $\left(\dfrac{1}{2}\right)^{(\frac{8}{2})}=\dfrac{1}{16}\neq \dfrac{1}{8}$ ❌
(5) $\left(\dfrac{1}{2}\right)^{(\frac{10}{2})}=\dfrac{1}{32}\neq \dfrac{1}{10}$ ❌
故選(2)

2.

如圖,OABC−DEFG 為一正方體,試問向量外積 $\overrightarrow{AD}\times \overrightarrow{AG}$ 與下列哪一個向量平行?

(1) $\overrightarrow{AE}$
(2) $\overrightarrow{BE}$
(3) $\overrightarrow{CE}$
(4) $\overrightarrow{DE}$
(5) $\overrightarrow{OE}$

(5)

將正方體座標化,如圖所示。
可得 $\begin{aligned}\overrightarrow{AD}\times \overrightarrow{AG}&=(0,1,1)\times (1,1,1)\\&=(0,-1,1)\end{aligned}$ 。算出選項中五個向量,找出與向量 $ (0,-1,1)$ 平行的選項即可。
(1) $\overrightarrow{AE}=(0,0,1)$ ❌
(2) $\overrightarrow{BE}=(-1,0,1)$ ❌
(3) $\overrightarrow{CE}=(-1,-1,1)$ ❌
(4) $\overrightarrow{DE}=(0,-1,0)$ ❌
(5) $\overrightarrow{OE}=(0,-1,1)$ ✔
故選(5)

3.

設 $a \in \{-6,-4,-2,2,4,6\}$ ,已知 a 為實係數三次多項式 $f(x)$ 的最高次項係數,若函數 $y=f(x)$ 的圖形與 $x$ 軸交於三點,且其 $x$ 坐標成首項為 $-7$、公差為 $a$ 的等差數列。試問共有幾個 $a$ 使得 $f(0)>0$?

(1) 1個

(2) 2個

(3) 3個

(4) 4個

(5) 5個

(1)

由題意可知此函數與 $x$ 軸交於 (-7,0)(-7+a,0)(-7+2a,0) ,故可設

\begin{aligned}f(x)&=a(x-(-7))(x-(-7+a))(x-(-7+2a))\\&=a(x+7)(x+7-a)(x+7-2a)\end{aligned}

\begin{aligned} f(0)&=7a(7-a)(7-2a)\\&=7a(a-7)(2a-7)\end{aligned}

7a(a-7)(2a-7)>0 ,可得 0<a<\dfrac{7}{2}a>7 (如下圖)

只有 $a=2$ 在範圍中,故選(1)

4.

試問有多少個實數 $x$ 滿足 \sin(x+\dfrac{\pi}{6})=\sin{x}+\sin{\dfrac{\pi}{6}}

(1) 1個

(2) 2個

(3) 3個

(4) 4個

(5) 5個(含)以上

(2)

\begin{aligned}&\sin(x+\dfrac{\pi}{6})=\sin{x}+\sin{\dfrac{\pi}{6}}\\ \Rightarrow &\sin{x}\cos{\dfrac{\pi}{6}}+\cos{x}\sin{\dfrac{\pi}{6}}=\sin{x}+\dfrac{1}{2}\\ \Rightarrow &\dfrac{\sqrt{3}}{2}\sin{x}+\dfrac{1}{2}\cos{x}-\sin{x}=\dfrac{1}{2}\\ \Rightarrow &(\sqrt{3}-2)\sin{x}+\cos{x}=1\end{aligned}
設 $a=\sqrt{(\sqrt{3}-2)^2+1^2}>1$

經過三角疊合後,可得:
a\sin(x+\theta)=1
即找
\left\{\begin{array}{l} y=a\sin(x+\theta)\\ y=1\end{array}\right. 的交點個數。
由於 a>1 (振幅大於1) ,可畫出下圖:

故有兩個交點(因為週期是 $2\pi$ ,一個完整週期就是兩個交點),(2)

5.

將1到50這50個正整數平分成甲乙兩組,每組各 25 個數,使得甲組的中位數比乙組的中位數小 1。試問共有幾種分法?

(1) $C^{50}_{25}$

(2) $C^{48}_{24}$

(3) $C^{24}_{12}$

(4) $(C^{24}_{12})^2$

(5) $C^{48}_{24}\cdot C^{24}_{12}$

(4)

假設甲的中位數是 $k$ ,乙的中位數是 $k+1$ ,將兩組的數由小到大排,畫出下圖:

由此可知,小於 $k$ 的數總共有 $12+12=24$ 個,所以 $k=25$
所以此題相當於 $1~24$ 共24個數中選12個放入甲,剩下12個放入乙。
並且 $27~50$ 共24個數中選12個放入甲,剩下12個放入乙。
故總方法數為 $C^{24}_{12}\times C^{24}_{12}=(C^{24}_{12})^2$
故選(4)

6.

在同一平面上,相距 7 公里的 $A,B$ 兩砲台, $A$ 在 $B$ 的正東方。某次演習時, $A$ 向西偏北 $\theta$ 方向發射砲彈, $B$ 則向東偏北 $\theta$ 方向發射砲彈,其中 $\theta$ 為銳角,觀測回報兩砲彈皆命中 9 公里外的同一目標 $P$。接著$A$ 改向西偏北 $\dfrac{\theta}{2}$ 方向發射砲彈,彈著點為 9 公里外的點 $Q$ 。試問砲台 $B$ 與彈著點 $Q$的距離 $\overline{BQ}$ 為何?

(1) 4公里

(2) 4.5公里

(3) 5公里

(4) 5.5公里

(5) 6公里

(2)

如上圖,只要能算出 $\cos{\frac{\theta}{2}}$ 的值,就可以利用餘弦定理算出 $\overline{BQ}$ 。
因為 $\begin{aligned}\cos\theta&=\dfrac{\overline{AC}}{\overline{AH}}\\&=\dfrac{3.5}{9}\\&=\dfrac{7}{18}\end{aligned}$
利用半角公式, $\begin{aligned}\cos{\frac{\theta}{2}}=\sqrt{\dfrac{1+\cos\theta}{2}}=\sqrt{\dfrac{25}{36}}=\dfrac{5}{6}\end{aligned}$
故由餘弦定理:
$\begin{aligned}\cos{\frac{\theta}{2}}&=\dfrac{ 9^2+7^2-\overline{BQ}^2}{ 2\times 9\times7 }\\ \Rightarrow \overline{BQ}&=5\end{aligned}$
故選(3)

二、多選題(占30分)

7.

令坐標平面上滿足 $ y=\log{x}$ 的點 $(x,y)$ 所成圖形為 $\Gamma$ ,試問滿足下列哪些關係式的 $(x,y)$ 所成圖形與 $\Gamma$ 完全相同?

(1) $y+\dfrac{1}{2}=\log(5x)$
(2) $2y=\log(x^2)$
(3) $3y=\log(x^3)$
(4) $x=10^y$
(5) $x^3=10^{(y^3)}$

(3)(4)

(1)

$\begin{aligned}&y+\dfrac{1}{2}=\log(5x) \\ \Rightarrow &y=\log{5x}-\log{10^{\frac{1}{2}}}\\ \Rightarrow &y=\log{\dfrac{5x}{\sqrt{10}}}\neq\log{x}\end{aligned}$

(2)

$\begin{aligned}&2y=\log(x^2) \\ \Rightarrow &y=\dfrac{1}{2}\log(x^2) \\ \Rightarrow & y=\log|x| \end{aligned}$

圖形如下:

(3)

$\begin{aligned}&3y=\log(x^3) \\ \Rightarrow &y=\dfrac{1}{3}\log(x^3) \\ \Rightarrow & y=\log{x} \end{aligned}$

(4)

由於 $x,10^y$ 皆比零大, 兩邊取$\log$
$\begin{aligned}\Rightarrow &\log{x}=\log{10^y}\\ \Rightarrow &y=\log{x}\end{aligned}$

(5)

$\begin{aligned}\Rightarrow &x^3=10^{(y^3)}\\ \Rightarrow &x=10^{\frac{1}{3}y^3}\end{aligned}$
與選項(4)比較,顯然不同。❌
(如果題目改為 $x^3=(10^y)^3$ 就會一樣)

故選(3)(4)

8.

對任一正整數 $n \geq 2$ ,令 $T_n$ 表示邊長為 $n, n+1,n+2$ 的三角形。試選出正確的選項。(註:若三角形的三邊長分別為 $a,b,c$ ,令 $s=\dfrac{a+b+c}{2}$,則三角形面積為 $\sqrt{s(s-a)(s-b)(s-c)}$)

(1) $T_n$ 皆為銳角三角形
(2) $T_2, T_3,T_4,…, T_{10}$ 的周長形成等差數列
(3) $T_n$ 的面積隨 $n$ 增大而增大
(4) $T_5$ 的三高依序形成等差數列

(5) $T_3$ 的最大角大於 $T_2$ 的最大角

(3)(5)

(1)

顯然 $n=3$ 時三邊長為 $3,4,5$ ,為直角三角形,所以不對。
事實上,只要計算最大角的 $\cos$ 值,如果是的,就是銳角;如果是的,就是頓角;如果是 $0$,那就是直角。
因此
$\begin{aligned}\cos\theta&=\dfrac{n^2+(n+1)^2-(n+2)^2}{2\times n\times (n+1)}\\&=\dfrac{n-3}{2n}\end{aligned}$
$\begin{aligned}{\color{red}銳角}\Rightarrow&\dfrac{n-3}{2n}>0\\ \Rightarrow&{\color{red}n>3}\end{aligned}$
$\begin{aligned}{\color{red}鈍角}\Rightarrow&\dfrac{n-3}{2n}<0\\ \Rightarrow&{\color{red}0<n<3}\end{aligned}$
$\begin{aligned}{\color{red}直角}\Rightarrow&\dfrac{n-3}{2n}=0\\ \Rightarrow&{\color{red}n=3}\end{aligned}$

(2)

$T_n$ 的周長為 $n+(n+1)+(n+2)=3n+3$
$\begin{aligned}後項-前項&=T_{n+1}-T_{n} \\&=[3(n+1)+3]-[3n+3] \\&=3\end{aligned}$
為定值,故 $T_2, T_3,T_4,…, T_{10}$ 的周長為等差數列。✔

(3)

$s=\dfrac{n+(n+1)+(n+2)}{2}=\dfrac{3n+3}{2}$ ,面積 $\begin{aligned}&=\sqrt{\dfrac{3n+3}{2}\times \left(\dfrac{3n+3}{2}-n\right)\times\left(\dfrac{3n+3}{2}-(n+1)\right)\times \left(\dfrac{3n+3}{2}-(n+2)\right)}\\&=\sqrt{\dfrac{3n+3}{2}\times\dfrac{n+3}{2}\times\dfrac{n+1}{2}\times\dfrac{n-1}{2}}\end{aligned}$
當 $n>1$ 時遞增,故正確。✔

(4)

只要找出有一個 $n$ 不符合,此選項就不對。
設 $n=2$ ,三邊長為 $2,3,4$ ,則高的比為 $\dfrac{1}{2}:\dfrac{1}{3}:\dfrac{1}{4}$ ,然而 $\dfrac{1}{3}-\dfrac{1}{2}=-\dfrac{1}{6}$ , $\dfrac{1}{4}-\dfrac{1}{3}=-\dfrac{1}{12}$ ,兩者不相等。故錯誤。❌

(5)

$T_3$ 為直角三角形, $T_2$ 為鈍角三角形。所以 $T_2$ 的最大角較大。❌

9.

某實驗室蒐集了大量的 $A$ 、 $B$ 兩相似物種,記錄其身長為 $x$ (單位:公分)與體重 $y$(單位:公克),得 $A$ 、 $B$ 兩物種的平均身長分別為 $\overline{x_A}=5.2$ 、 $\overline{x_B}=6$ ,標準差分別為 0.3、0.1。令 $A$ 、 $B$ 兩物種的平均體重分別為 $\overline{y_A}$ 、 $\overline{y_B}$ 。若 $A$ 、 $B$ 兩物種其體重 $y$ 對身長 $x$ 的迴歸直線分別為 $L_A: y=2x-0.6$ 、 $L_B:y=1.5x+0.4$ ,相關係數分別為 0.6、0.3。今發現一隻身長 5.6 公分、體重 8.6 公克的個體 $P$ ,試選出正確的選項。

(1) $\overline{y_A}<\overline{y_B}$
(2) $A$ 物種的體重標準差小於 $B$ 物種的體重標準差
(3) 就 $A$ 物種而言,個體 $P$ 的體重與平均體重 $\overline{y_A}$ 之差的絕對值大於一個標準差

(4) 點 $(5.6, 8.6)$ 到直線 $\overline{L_A}$ 的距離小於其到直線 $\overline{L_B}$的距離

(5) 點 $(5.6, 8.6)$ 與點 $(\overline{x_A},\overline{y_A})$ 的距離小於其與點 $(\overline{x_B},\overline{y_B})$ 的距離

(3)

本題重點:回歸直線的斜率為 $ r\dfrac{\sigma_y}{\sigma_x}$ ,且此直線會通過 $(\mu_x,\mu_y) $

(1)

將 $(\overline{x_A},\overline{y_A}) $ 代入 $y=2x-0.6$ 可得 $\overline{y_A}=9.8\ $ ; $(\overline{x_B},\overline{y_B}) $ 代入 $y=1.5x+0.4$ 可得 $\overline{y_B}=9.4$ ,所以 $\overline{y_A}>\overline{y_B}$ ❌

(2)

$L_A$ 斜率 $=2=r_A\times\dfrac{\sigma_{y_A}}{\sigma_{x_A}}$ $\Rightarrow \sigma_{y_A}=1$
$L_B$ 斜率 $=2=r_B\times\dfrac{\sigma_{y_B}}{\sigma_{x_B}}$ $\Rightarrow \sigma_{y_B}=0.5$
所以 $\sigma_{y_A}>\sigma_{y_B}$ ❌

(3)

$|8.6-\overline{y_A}|=|8.6-9.8|=1.2>1$ ✔

(4)

$(5.6, 8.6)$ 到直線 $\overline{L_A}$ 的距離 $=\dfrac{|2\times 5.6-8.6-0.6|}{\sqrt{2^2+1^2}}=\dfrac{2}{\sqrt{5}}$
$(5.6, 8.6)$ 到直線 $\overline{L_B}$ 的距離 $=\dfrac{|1.5\times 5.6-8.6+0.4|}{\sqrt{1.5^2+1^2}}=\dfrac{0.2}{\sqrt{3.25}}=\dfrac{2}{5\sqrt{13}}$
因為 $\dfrac{2}{\sqrt{5}}>\dfrac{2}{5\sqrt{13}}$ ,所以 $(5.6, 8.6)$ 到直線 $\overline{L_A}$ 大於 $(5.6, 8.6)$ 到直線 $\overline{L_B}$ 的距離。❌

(5)

由於 $(\overline{x_A},\overline{y_A})=(5.2,9.8)$ , $(\overline{x_B},\overline{y_B})=(6,9.4)$ ,兩者與 $(5.6, 8.6)$ 的距離分別為 $\sqrt{0.4^2+1.2^2}$ 與 $\sqrt{0.4^2+0.8^2}$ ,前者較大。❌

故選(3)

10.

坐標平面上有一正方形與一正六邊形,正方形在正六邊形的右邊。已知兩正多邊形都有一邊在 $x$ 軸上,且正方形中心 $A$ 與正六邊形中心 $B$ 都在 $x$ 軸的上方,且兩多邊形恰有一個交點 $P$ ,又知正方形的邊長為 6,而點 $P$ 到 $x$ 軸的距離為 $2\sqrt{3}$ 。試選出正確的選項。
(1) 點 $A$ 到 $x$ 軸的距離大於點 $B$ 到 $x$ 軸的距離

(2) 正六邊形的邊長為 6
(3) $ \overrightarrow{BA}=(7,3-2\sqrt{3})$
(4) $\overline{AP}>\sqrt{10}$
(5) 直線 $AP$ 斜率大於 $-\dfrac{1}{\sqrt{3}}$

(3)(5)

依照題意將圖畫出,如下圖。

由於 $P$ 到 $x$ 軸距離為 $2\sqrt{3}$ ,且正六邊形的外角是 $60^\circ$ ,所以正六邊形邊長為 $2\sqrt{3}\times\dfrac{2}{\sqrt{3}}=4$
因此可求出 $A$ 、 $B$ 、 $P$ 三點座標: $A(9,3),B(2,\sqrt{3}),P(6,2\sqrt{3})$
(注意:這取決於 $y$ 軸定在哪裡,不會影響到最後答案。)

(1)

$A$ 到 $x$ 軸的距離 $=3$
$B$ 到 $x$ 軸的距離 $=2\sqrt{3}$
後者較大。❌

(2)

邊長為 $4$ ❌

(3)

$\overrightarrow{BA}=(9-2,3-2\sqrt{3})=(7,3-2\sqrt{3})$ ✔

(4)

$\overline{AP}=\sqrt{(9-6)^2+(3-2\sqrt{3})^2}=\sqrt{9+(3-\sqrt{12})^2}<\sqrt{9+1}=\sqrt{10}$
(因為 $\sqrt{12}=3.abc…$ ,所以 $(3-\sqrt{12})^2<1$)
因此 $\overline{AP}<\sqrt{10}$ ❌

(5)

直線 $AP$ 斜率 $=\dfrac{2\sqrt{3}-3}{6-9}=\dfrac{3-2\sqrt{3}}{3}$
分子分母同除以 $\sqrt{3}\Rightarrow\dfrac{\sqrt{3}-2}{\sqrt{3}}$
因為 $2-\sqrt{3}<1$ ,所以 $\sqrt{3}-2>-1$
故 $\dfrac{\sqrt{3}-2}{\sqrt{3}}>-\dfrac{1}{\sqrt{3}}$
因此直線 $AP$ 斜率大於 $-\dfrac{1}{\sqrt{3}}$ 。✔

故選(3)(5)

11.

考慮二元一次方程組 \left\{\begin{array}{l} ax+6y=6\\x+by=1\end{array}\right. ,其係數 $a,b$ 之值分別由投擲一顆公正骰子與一枚均勻硬幣來決定。令 $a$ 值為骰子出現之點數;若硬幣出現正面時 $b$ 值為 $1$ ,若硬幣出現反面時 $b$ 值為 $2$。試選出正確的選項。

(1) 擲出 $a=b$ 的機率為 $\dfrac{1}{3}$

(2) 此方程組無解的機率為 $\dfrac{1}{12}$

(3) 此方程組有唯一解的機率為 $\dfrac{5}{6}$

(4) 硬幣出現反面且此方程組有解的機率為 $\dfrac{1}{2}$

(5) 在硬幣出現反面且此方程組有解的條件下, $x$ 值為正的機率為 $\dfrac{2}{5}$

(2)(3)

樣本空間的個數共有 $6\times 2=12$

(1)

只有 $(a,b)=(1,1)$ 、 $(2,2)$ 兩種可能。所以機率為 $\dfrac{2}{12}=\dfrac{1}{6}$ ❌

(2)

當 $\dfrac{a}{1}=\dfrac{6}{b}\neq\dfrac{6}{1}$ 時,方程組無解。
由於 $b$ 只有 $1,2$ 兩種可能,所以分別討論即可。
$b=1$ 時, $a=6$ ,但是 $\dfrac{6}{b}=6$ (此時無限多組解)
$b=2$ 時, $a=3$ ,此時無解。
所以只有一種可能。機率為 $\dfrac{1}{12}$ ✔

(3)

把所有可能扣掉無解與無限多組解,剩下就是唯一解。所以機率為 $1-\dfrac{1}{12}-\dfrac{1}{12}=\dfrac{5}{6}$ ✔

(4)

反面代表 $b=2$ ,有解代表 $a\neq 3$
所以 $(a,b)$ 只有 $(2,1),(2,2),(2,4),(2,5),(2,6)$ 五種可能。機率為 $\dfrac{5}{12}$ ❌

(5)

假設A代表硬幣出現反面且此方程組有解的情況
B代表 $x$ 值為正的情況
則題目要求的是 $P(B|A)=\dfrac{P(A\cap B)}{P(A)}$
將 $b=2$ 帶入原式:
$\left\{\begin{array}{l} ax+6y=6\\x+2y=1\end{array}\right.$ $\Rightarrow x=\dfrac{3}{a-3}$ ,當 $a>3$ 時, $x$ 值為正。所以只有 $(a,b)=(4,2),(5,2),(6,2)$ 三種可能。
因此 $P(A\cap B)=\dfrac{3}{12}$ ,故 $P(B|A)=\dfrac{\frac{3}{12}}{\frac{5}{12}}=\dfrac{3}{5}$ ❌

故選(2)(3)

12.

在坐標平面上給定三點 $A(1,0)$ 、 $B(0,1)$ 、 $C(-1,0)$ ,令 $\Gamma$ 為 ΔABC經矩陣$\begin{bmatrix} 3 & 0\\ a& 1\end{bmatrix}$ 變換後的圖形,其中 $a$ 為實數。試選出正確的選項。
(1) 若 $a=0$ ,則 $\Gamma$ 為等腰直角三角形
(2)$\triangle ABC$ 的邊上至少有兩點經 $T$ 變換後坐標不變
(3) $\Gamma$ 必有部分落在第四象限
(4) 平面上找得到一個圖形 $\Omega$ 經 $T$ 變換後為 $\triangle ABC$
(5) $\Gamma$ 的面積為定值

(2)(4)(5)

(1)

$A’:\begin{bmatrix} 3 & 0\\ 0& 1\end{bmatrix}\begin{bmatrix} 1 \\ 0\end{bmatrix}=\begin{bmatrix} 3 \\ 0\end{bmatrix}$
$B’:\begin{bmatrix} 3 & 0\\ 0& 1\end{bmatrix}\begin{bmatrix} 0 \\ 1\end{bmatrix}=\begin{bmatrix} 0 \\ 1\end{bmatrix}$
$C’:\begin{bmatrix} 3 & 0\\ 0& 1\end{bmatrix}\begin{bmatrix} -1 \\ 0\end{bmatrix}=\begin{bmatrix} -3 \\ 0\end{bmatrix}$

如圖, $\triangle A’B’C’$ 並非等腰「直角」三角形。❌

(2)

$\begin{bmatrix} 3 & 0\\ a& 1\end{bmatrix}\begin{bmatrix} x\\y\end{bmatrix}=\begin{bmatrix} x\\y\end{bmatrix}$
$\Rightarrow \begin{bmatrix} 3x\\ax+y\end{bmatrix}=\begin{bmatrix} x\\y\end{bmatrix}$
解得 $ \left\{\begin{array}{l} x=0\\y=y\end{array}\right.$
所以只要在 $y$ 軸上的點經過變換後座標都不會變。
$\triangle ABC$ 的邊上有兩個點在 $y$ 軸上,所以至少有兩個點經過變換後座標不變。✔

(3)

不一定,例如在選項(1)中,沒有任何點在第四象限。❌

(4)

在(1)中我們知道經過 $\begin{bmatrix} 3 & 0\\ 0& 1\end{bmatrix}$ 變換後, $x$ 座標都會變成原本的三倍,所以只要找一個三角形,$x$ 座標變成三倍後與 $\triangle ABC$ 重合即可。(如圖)✔

(5)

$\begin{aligned}\triangle{A’B’C’}面積&=\det(\begin{bmatrix} 3 & 0\\ a& 1\end{bmatrix})\times\triangle{ABC}面積\\&=3\times\triangle{ABC}面積\end{aligned} $
其為定值。✔

故選(2)(4)(5)

三、選填題(占 25 分)

13.

某銷售站銷售甲、乙、丙三型手機,甲手機每支利潤 100 元,乙手機每支利潤 400 元,丙手機每支利潤 240 元。上年度甲、乙、丙手機各賣出 $A,B,C$ 支,平均每支利潤為 260 元;且知銷售甲、乙兩型手機共 $A+B$ 支的平均每支利潤為 280 元。則該站上年度售出的三型手機數量比為 $A:B:C= \underline{\color{red}\mbox{(13-1)}}: \underline{\color{red}\mbox{(13-2)}}:\underline{\color{red}\mbox{(13-3)}}$ 。(化為最簡整數比)

按照題意可列式:
$\left\{\begin{array}{l} \dfrac{100A+400B+240C}{A+B+C}=260\\ \dfrac{100A+400B}{A+B}=280\end{array}\right.$
將第二式整理 $\Rightarrow 180A=120B\Rightarrow A:B=2:3$
假設 $A=2k,B=3k$ 代入第一式 $\Rightarrow C=5k$
因此 $A:B:C=2:3:5$

14.

已知 $f(x)$ 、 $g(x)$ 、 $h(x)$ 皆為實係數三次多項式,且除以 $x^2-2x+3$ 的餘式分別為 $x +1$ 、 $ x -3$ 、 $-2$ 。若 $xf(x)+ag(x)+bh(x)$ 可以被 $x^2-2x+3$整除,其中 $a,b$為實數,則 $a=\underline{\color{red}\mbox{(14-1)}\mbox{(14-2)}}$ , $b=\underline{\color{red}\mbox{(14-3)}}$ 。


$f(x)=(x^2-2x+3)Q_1(x)+(x+1)$
$g(x)=(x^2-2x+3)Q_2(x)+(x-3)$
$h(x)=(x^2-2x+3)Q_3(x)+(-2)$

$\begin{aligned}xf(x)+ag(x)+bh(x)&=x{\color{red}(x^2-2x+3)}Q_1(x)+x(x+1)\\&+a{\color{red}(x^2-2x+3)}Q_2(x)+a(x-3) \\&+b{\color{red}(x^2-2x+3)}Q_3(x)+b(-2)\end{aligned}$
左半邊都能被 $x^2-2x+3$ 整除,所以 $x(x+1)+a(x+3)+b(-2)$ 可以被 $x^2-2x+3$ 整除。
$x(x+1)+a(x-3)+b(-2)=x^2+(a+1)x+(-3a-2b)$ ,利用長除法:

整除代表餘式為0,故 $\left\{\begin{array}{l} a=-3\\b=3\end{array}\right.$

15.

某商場舉辦現場報名的摸彩箱抽獎活動,報名截止後,主持人依報名人數置入同數量的摸彩球,其中有 10 顆被標示為幸運獎,其獎項為 5000 元禮券及 8000 元禮券各 5 顆,每顆球被抽中的機率皆相同,抽後不放回。抽獎前,主辦單位依獎項個數與報名人數,主持人公告中獎機率為 0.4%。開始抽獎後,每人依序抽球,每個人只有一次抽獎機會。若前 100 位參加抽獎者,恰有 1 人抽中 5000 元禮券且沒有人抽中 8000 元禮券,則抽獎順序為第 101 號者可獲禮券金額的期望值為$\underline{\color{red}\mbox{(15-1)}\color{red}\mbox{(15-2)}}$ 元

設共有 $x$ 人抽獎,則 $\dfrac{10}{x}=0.4\%$ ,解得 $x=\dfrac{10}{0.4\%}=2500$
第 $101$ 人抽獎時,剩下 $4$ 個 $5000$ 元、 $5$ 個 $8000$ 元。
所以獲得金額的期望值為 $\left(\dfrac{4}{2500-100}\right)\times{5000}+\left(\dfrac{5}{2500-100}\right)\times 8000=25$

16.

坐標平面上,已知向量 $\vec{v}\ $在向量 $(2,-3)$ 方向的正射影長比原長少 1,而在向量 $(3,2)$ 方向的正射影長比原長少 2。若 $\vec{v}\ $與兩向量 $(2, -3),(3,2)$ 的夾角皆為銳角,則 $\vec{v}\ $在向量 $(4,7)$ 方向的正射影長為 $\underline{\dfrac{{\color{red}{\mbox{(16-1)}}}\sqrt{\color{red}{\mbox{(16-2)}} }}{\color{red}{\mbox{(16-3)}}} }$ 。(化為最簡根式)

假設 $|\vec{v}|=t$ ,觀察 $(2,-3)\cdot (3,2)=0$ ,所以 $(2,-3)\perp (3,2)$ ,將圖形畫出來如下圖(示意圖,非正確比例):

由畢氏定理可得 $(t-1)^2+(t-2)^2=t^2$ ,解出 $t=5$ 或 $t=1$ (不合,因為 $t-2<0$)
因此 $\vec{v}=\dfrac{5-1}{|(2,-3)|}(2,-3)+\dfrac{5-2}{|(3,2)|}(3,2)=\left(\dfrac{17}{\sqrt{13}},\dfrac{-6}{\sqrt{13}}\right)$
故所求 $=\dfrac{\vec{v}\cdot (4,7)}{|(4,7)|}=\dfrac{\left(\dfrac{17}{\sqrt{13}},\dfrac{-6}{\sqrt{13}}\right)\cdot (4,7)}{|(4,7)|}=\dfrac{\dfrac{26}{\sqrt{13}}}{\sqrt{65}}=\dfrac{2}{\sqrt{5}}=\dfrac{2\sqrt{5}}{5}$

17.

坐標平面上,在以 $O(0,0), A(0,1) ,B(1,1), C(1,0)$ 為頂點的正方形(含邊界)內,令 $R$ 為滿足下述條件的點 $P(x,y)$ 所成區域:與點 $P(x,y)$ 的距離為 $|x-y|$ 之所有點所成圖形完全落在正方形 $OABC$ (含邊界)內。則區域 $R$ 的面積為 $\underline{\dfrac{\color{red}\mbox{(17-1)}}{ \color{red}\mbox{(17-2)} }}$ 。(化為最簡分數)

題目相當於問:收集所有滿足以下條件的 $P(x,y)$: $P(x,y)$ 為圓心, $|x-y|$ 為半徑的圓完全落在正方形內。且這些 $P$ 所形成的區域稱為 $R$ , $R$ 的面積
因為 $|x-y|$ 很難看出其數值,我們討論兩個情形: $x\geq y$ 以及 $x\leq y$ ,如此一來就能拆絕對值了。
當 $x\geq y$ 時(即正方形的右下半部),此時半徑為 $x-y$,為了讓圓完全落在正方形內:

  • $(x,y)$ 在 $x$ 方向加上半徑後,不能超過 $x=1$這條邊界。 $\Rightarrow x+(x-y)\leq 1\Rightarrow 2x-y\leq 1$
  • $(x,y)$ 在 $y$ 方向減掉半徑(因為下方才會超出邊界)後,不能超過 $y=0$ 這條邊界。
    $\Rightarrow y-(x-y)\geq 0\Rightarrow 2y\geq x$
    將解的範圍畫在圖上:

將 $2x-y=1$ 與 $2y=x$ 兩條直線的交點解出 $\left(\dfrac{2}{3},\dfrac{1}{3}\right)$
即可計算面積(這裡方法很多,可以使用測量師公式、向量..等等,這裡使用向量計算。)
故面積 $\dfrac{1}{2}\left |\left |\begin{array}{ccc}\dfrac{2}{3} &\dfrac{1}{3} \\1 &1 \end{array}\right|\right|=\dfrac{1}{6}$
同理 $x\geq y$ 時,面積也是 $\dfrac{1}{6}$ (也就是上面圖形以 $x=y$ 對稱。)
因此總面積為 $\dfrac{1}{6}+\dfrac{1}{6}=\dfrac{1}{3}$


第貳部分:混合題 或非選擇題(占 15 分)

18-20題為題組

坐標空間中,設 $O$ 為原點, $E$ 為平面 $x -z = 4$ 。試回答下列問題。

18.

若原點 $O$ 在平面 $E$ 上的投影點為 $Q$ ,且向量 $\overrightarrow{OQ}$ 與向量 $(1,0,0)$ 的夾角為 $\alpha$ ,則 $\cos\alpha$ 之值為下列哪一選項?(單選題,3 分)

(1) $-\dfrac{\sqrt{2}}{2}$

(2) $-\dfrac{1}{2}$

(3) $\dfrac{1}{2}$

(4) $\dfrac{\sqrt{2}}{2}$

(5) $\dfrac{\sqrt{2}}{2}$

因為 $\overrightarrow{OQ}\perp E$ ,所以 $\overrightarrow{OQ}$ 與 $E$ 的法向量平行,也就是 $\overrightarrow{OQ}//(1,0,-1)$
假設 $\overrightarrow{OQ}=(t,0,-t)$,代表 $Q$ 點座標為 $(t,0,-t)$ ,代入平面可得 $2t=4\Rightarrow t=2$ ,因此 $\overrightarrow{OQ}=(2,0,-2)$
故 $\cos\alpha=\dfrac{\overrightarrow{OQ}\cdot (1,0,0)}{|\overrightarrow{OQ}||(1,0,0)|}=\dfrac{2}{2\sqrt{2}}=\dfrac{\sqrt{2}}{2}$ ,故選(4)

19.

已知空間中有一點 $P(a,b,c)$ 滿足向量 $ \overrightarrow{OP}$ 與向量 $(1,0,0)$ 的夾角 $\theta \leq\dfrac{\pi}{6}$ 。試說明實數 $a,b,c$ 滿
足不等式 $a^2\geq 3(b^2+c^2)$ 。(非選擇題,4 分)

由題意可知 $ \overrightarrow{OP}=(a,b,c)$ ,所以 $\cos\theta=\dfrac{(a,b,c)\cdot (1,0,0)}{|(a,b,c)||(1,0,0)|}=\dfrac{a}{\sqrt{a^2+b^2+c^2}}$
因為 $\theta\leq\dfrac{\pi}{6}\Rightarrow \cos\theta\geq \dfrac{\sqrt{3}}{2}$ , 因此
$\begin{aligned}\dfrac{a}{\sqrt{a^2+b^2+c^2}}\geq\dfrac{\sqrt{3}}{2}&\Rightarrow 2a\geq\sqrt{3a^2+3b^2+3c^2}\\&\Rightarrow 4a^2\geq 3a^2+3b^2+3c^2\\&\Rightarrow a^2\geq 3(b^2+c^2)\end{aligned}$

20.

承 19 題,已知點 $P$ 在平面 $E$ 上且 $b = 0$ 。試求 $c$ 的最大可能範圍,並求線段 $\overline{OP}$ 的最小可能長度。(非選擇題,8 分)

因為 $b=0$ ,代入19題的結果 $\Rightarrow a^2\geq 3c^2$
因為 $P$ 在平面 $E$ 上, $(a,0,c)$ 代入平面方程式會成立 $\Rightarrow a-c=4\Rightarrow a=c+4$
將第二式代入第一式 $\Rightarrow 3c^2-(c-4)^2\leq 0\Rightarrow 2c^2+8c-16\leq 0\Rightarrow 2-2\sqrt{3}\leq c\leq 2+2\sqrt{3}$ 。即為 $c$ 的最大可能範圍。
$\overline{OP}=\sqrt{a^2+c^2}$
將 $a=c+4$ 代入 $\Rightarrow \overline{OP}=\sqrt{(c+4)^2+c^2}=\sqrt{2c^2+8c+16}=\sqrt{2(c+2)^2+8}$ ,因為根號內是一個開口向上的拋物線,所以 $c$ 越接近 $-2$ 得到的值會越小。在 $2-2\sqrt{3}\leq c\leq 2+2\sqrt{3}$ 的範圍中, $c=2-\sqrt{3}$ 最接近,故代入得
$\begin{aligned}\sqrt{2(2-2\sqrt{3}+2)^2+8}&=\sqrt{64-32\sqrt{3}}\\&=\sqrt{64-2\sqrt{768}}\\&=\sqrt{64-2\sqrt{48\times 16}}\\&=\sqrt{(\sqrt{48}-\sqrt{16})^2}\\&=4\sqrt{3}-\sqrt{4}\end{aligned}$



資料來源:大考中心

發佈留言

發佈留言必須填寫的電子郵件地址不會公開。 必填欄位標示為 *